Convergent Series: sin(kx)/ln(k)

  • Thread starter Thread starter ForMyThunder
  • Start date Start date
  • Tags Tags
    Convergent Series
Click For Summary
The series \(\sum^{\infty}_{k=2}\frac{sin(kx)}{ln(k)}\) is being analyzed for convergence for all real x. It is noted that the series is bounded, as \(\frac{1}{ln(2)}\sum^{\infty}_{k=2}sin(kx)\) is less than \(\frac{\pi}{x ln2}\). The Dirichlet Test is considered for convergence, despite concerns about the behavior of \(\sin(kx)\) not converging to zero. The discussion highlights that if \(x\) is not a rational multiple of \(2\pi\), the sequence of partial sums of \(\sin(kx)\) may still be bounded, supporting the argument for convergence. Ultimately, the boundedness of \(\sum sin(kx)\) is affirmed, reinforcing the case for the series' convergence.
ForMyThunder
Messages
149
Reaction score
0

Homework Statement



Is this series convergent for all real x:

\sum^{\infty}_{k=2}\frac{sin(kx)}{ln(k)}

Homework Equations


The Attempt at a Solution



This series is less than

\frac{1}{ln(2)}\sum^{\infty}_{k=2}sin(kx)

which is less than \frac{\pi}{x ln2}. So, the series is bounded for all x. I'm thinking that the Dirichlet Test would show that this series converges.
 
Last edited:
Physics news on Phys.org
Er, but sin(kx) doesn't converge to zero as k goes to infinity.
 
But the sum of the sin(kx) would still be bounded, right?

If you mean the sum of sin(kx), I found that it was less than 2pi/x in this way:

Consider the interval [0,2pi] with x in this interval. Then there are at most 2pi/x terms, with some of them being < 0. Since sin(y) <= 1 for all y, we have that the sum of the |sin(kx)| < 2pi/x.

Since the partial sums of sin(kx) is bounded and 1/ln(k) is a nonincreasing nullsequence, would this mean that the the series converges by Dirichlet's Test?
 
Last edited:
Then there are at most 2pi/x terms
The sum you wrote is over all k, not just those k for which 0 \leq kx \leq 2\pi.



If x is not a rational multiple of 2pi, I'm not sure if I expect the sequence of partial sums of sin(kx) to be bounded or not. But if that sequence is bounded, then your argument is valid.
 
Take a look at Example 1, under corollary 6, in this PDF: http://people.oregonstate.edu/~peterseb/mth311/docs/311abel.pdf

And yes you're right, \sum sin(kx) is bounded.
 
Last edited by a moderator:
Question: A clock's minute hand has length 4 and its hour hand has length 3. What is the distance between the tips at the moment when it is increasing most rapidly?(Putnam Exam Question) Answer: Making assumption that both the hands moves at constant angular velocities, the answer is ## \sqrt{7} .## But don't you think this assumption is somewhat doubtful and wrong?

Similar threads

  • · Replies 5 ·
Replies
5
Views
2K
  • · Replies 7 ·
Replies
7
Views
2K
  • · Replies 3 ·
Replies
3
Views
1K
  • · Replies 2 ·
Replies
2
Views
1K
  • · Replies 2 ·
Replies
2
Views
2K
  • · Replies 3 ·
Replies
3
Views
2K
  • · Replies 1 ·
Replies
1
Views
2K
  • · Replies 1 ·
Replies
1
Views
2K
Replies
8
Views
2K